LSAT and Law School Admissions Forum

Get expert LSAT preparation and law school admissions advice from PowerScore Test Preparation.

 Administrator
PowerScore Staff
  • PowerScore Staff
  • Posts: 8917
  • Joined: Feb 02, 2011
|
#38551
Complete Question Explanation
(The complete setup for this game can be found here: lsat/viewtopic.php?t=15133)

The correct answer choice is (B)

If N is published in the fall, then K must also be published in the fall (second rule). By the contrapositive of the fourth rule, M must be published in the spring. O and P must both be published in the fall (third and first rules, respectively).

These deductions can easily be made using the chain relationships discussed above, and can automatically be inferred from Template 1:
PT76_O15 LG Explanations_Game #3_#21_diagram 1.png
These inferences eliminate every single answer choice except for answer choice (B), which is correct. Observant test-takers, however, would have chosen that answer choice in a split second, without doing any of the work we just did. Why? Well, if the question asks what could be true, and we know that L is random and can therefore be placed in either group regardless of the placement of any of the other variables, then answer choice (B) should be clear.
You do not have the required permissions to view the files attached to this post.

Get the most out of your LSAT Prep Plus subscription.

Analyze and track your performance with our Testing and Analytics Package.